Đến nội dung

Nguyenhuyen_AG

Nguyenhuyen_AG

Đăng ký: 09-09-2010
Offline Đăng nhập: 10-01-2019 - 16:22
****-

#566840 Topic tổng hợp một số bất đẳng thức trong kì thi MO các nước

Gửi bởi Nguyenhuyen_AG trong 19-06-2015 - 13:01

Bài 116 (CĐTMO 2005) : Chứng minh rằng

                                                                 $\frac{a^{3}}{(b+c)^{3}}+\frac{b^{3}}{(c+a)^{3}}+\frac{c^{3}}{(a+b)^{3}}$

                                        trong đó $a,b,c$ là các số dương.

 

Đề bài này sai rồi, đề đúng là như vầy

\[\frac{a^{3}}{(a+b)^{3}}+\frac{b^{3}}{(b+c)^{3}}+\frac{c^{3}}{(c+a)^{3}} \geqslant \frac{3}{8}.\]

Tác giả là thầy Nam Dũng.

 

Lời giải 1. Sử dụng bất đẳng thức Cauchy-Schwarz, ta có \[\left [ \sum \left ( \frac{a}{a+b} \right )^3 \right ]\left [ \sum c^3(a+b)^3 \right ] \ge \left (\sum c^{\frac{3}{2}}a^{\frac{3}{2}} \right )^2,\] như vậy ta cần chứng minh được \[8\left (\sum c^{\frac{3}{2}}a^{\frac{3}{2}}\right )^2 \ge 3 \sum c^3(a+b)^3 .\] Đặt $x=\sqrt{ab},\,y=\sqrt{bc},\,z=\sqrt{ca},$ bất đẳng thức trên trở thành \[8\left (\sum z^3\right )^2 \ge 3 \sum (y^2+z^2)^3,\] hay là \[\sum (x^6+y^6)+16\sum x^3y^3 \ge 9 \sum x^2y^2(x^2+y^2),\] bất đẳng thức này đúng vì \[x^6+y^6+16x^3y^3-9x^2y^2(x^2+y^2) =(x-y)^4(x^2+4xy+y^2) \ge 0.\] Chứng minh hoàn tất.

 

Lời giải 2. Theo bất đẳng thức trung bình lũy thừa, ta có \[\sqrt[3]{\frac{\displaystyle\sum \left ( \frac{a}{a+b} \right )^3}{3}}\ge\sqrt{\frac{\displaystyle\sum \left ( \frac{a}{a+b} \right )^2}{3}}.\] Như vậy, để hoàn tất chứng minh thì ta cần chỉ ra được \[\left ( \frac{a}{a+b} \right )^2+\left ( \frac{b}{b+c} \right )^2+\left ( \frac{c}{c+a} \right )^2 \ge \frac{3}{4}.\] Thay $\left ( \frac{b}{a},\,\frac{c}{b},\,\frac{a}{c} \right )$ bởi $(x,\,y,\,z)$ ta sẽ đưa bài toán về chứng minh \[\frac{1}{(x+1)^2}+\frac{1}{(y+1)^2}+\frac{1}{(z+1)^2}\ge\frac{3}{4},\] với điều kiện $xyz=1.$

 

Sử dụng bất đẳng thức Cauchy-Schwarz, ta có \[\frac{1}{(x+1)^2}+\frac{1}{(y+1)^2}\ge \frac{1}{(\frac{x}{y}+1)(xy+1)}+\frac{1}{(\frac{y}{x}+1)(xy+1)}=\frac{1}{xy+1}.\] Ta sẽ chứng minh \[\frac{1}{xy+1}+\frac{1}{(z+1)^2}\ge \frac{3}{4},\] hay là \[\frac{z}{1+z}+\frac{1}{(z+1)^2}\ge \frac{3}{4}.\] Bất đẳng thức này đúng bởi vì \[\frac{z}{1+z}+\frac{1}{(z+1)^2}-\frac{3}{4}=\frac{(z-1)^2}{4(z+1)} \ge 0.\]
Nhận xét. Một cách khác để chứng minh bất đẳng thức \[\left ( \frac{a}{a+b} \right )^2+\left ( \frac{b}{b+c} \right )^2+\left ( \frac{c}{c+a} \right )^2 \ge \frac{3}{4}.\] Sử dụng bất đẳng thức Cauchy-Schwarz, ta có \[\left [ \sum \left ( \frac{a}{a+b} \right )^2 \right ]\left [ \sum (a+b)^2(a+c)^2 \right ]\ge \left [ \sum a(a+c) \right ]^2.\] Tuy nhiên \[4\left [ \sum a(a+c) \right ]^2 = \left [ \sum (a+b)^2 \right ]^2\] Vậy, ta chỉ cần chứng minh \[\left [ \sum (a+b)^2 \right ]^2 \ge 3 \sum (a+b)^2(a+c)^2,\] nhưng bất đẳng thức này đúng theo bất đẳng thức cơ bản \[(x+y+z)^2\ge 3(xy+yz+zx).\] Bất đẳng thức  được chứng minh.

 

Ngoài ra ta có thể chứng minh bài toán bằng cách sử dụng bất đẳng thức China TST 2004 của giáo sư Vasile Cirtoaje

\[\frac{a^2}{(a+b)^2}+\frac{b^2}{(b+c)^2}+\frac{c^2}{(c+d)^2}+\frac{d^2}{(d+a)^2} \geqslant 1.\]

 

 

Bài 115 (CĐTMO 2006) : Chứng minh rằng với mọi số thực $x,y,z$ thuộc đoạn $[1;2]$ ta luôn có bất đẳng thức :
                                          $(x+y+z)(\frac{1}{x}+\frac{1}{y}+\frac{1}{z})\geq 6(\frac{x}{y+z}+\frac{y}{x+z}+\frac{z}{x+y}).$

                                           Hỏi đẳng thức xảy ra khi và chỉ khi nào ?
 

 

Đặt \[f(x,\,y,\,z)=(x+y+z)\left( \frac{1}{x}+\frac{1}{y}+\frac{1}{z} \right)-6\left( \frac{x}{y+z}+\frac{y}{z+x}+\frac{z}{x+y} \right),\]

ta sẽ chứng minh $f(x,\,y,\,z)\ge 0$ bằng kỹ thuật dồn biến. Thật vậy, ta có $f(x,\,y,\,z)-f\left ( x,\,\frac{y+z}{2},\, \frac{y+z}{2}\right ),$ sẽ bằng \[\frac{(x+y+z)(y-z)^2}{yz(y+z)}-\frac{6(x+y+z)(y-z)^2}{(x+y)(x+z)(2x+y+z)},\] hay \[\frac{(x+y+z)(y-z)^2[(x+y)(z+x)(2x+y+z)-6yz(y+z)]}{(x+y)(y+z)(z+x)(2x+y+z)}.\] Giả sử $x$ là số lớn nhất trong ba số thì $(x+y)(z+x)(2x+y+z) > 6yz(y+z),$ nên $f(x,\,y,\,z)\ge f(x,\,t,\,t)$ với $t=\frac{y+z}{2} \ge 1.$

 

Tiếp đến, ta sẽ chứng minh $f(x,\,t,\,t)\ge 0.$ Bất đẳng thức tương đương với

$$(x+2t)\left( \frac{1}{x}+\frac{2}{t} \right)-6\left( \frac{x}{2t}+\frac{2t}{t+x} \right)\ge 0,$$

$$\frac{{{(t-x)}^{2}}(2t-x)}{tx(t+x)}\ge 0,$$

hiển nhiên đúng vì $2t\ge 2\ge x.$ Đẳng thức xảy ra khi và chỉ khi $x=y=z,$ hoặc $x=2$ và $y=z=1.$ Bài toán được chứng minh.

 

Nhận xét. Nếu $x,\,y,\,z$ là độ dài ba cạnh của tam giác thì bài toán vẫn đúng. Thật vậy, vì $x,\,y,\,z$ là độ dài ba cạnh của tam giác nên, theo phép thế Ravi, ta có thể viết bất đẳng thức lại như sau \[(x+y+z)\sum \frac{1}{x+y} \ge 3\sum \frac{y+z}{2x+y+z},\] tương đương với \[2\sum x^2(x^2-y^2)(x^2-z^2)+3\sum yz(y^2-z^2)^2 \ge 0,\] hiển nhiên đúng theo bất đẳng thức Shur.

 

Tác giả bài toán cũng là thầy Nam Dũng.




#566423 Topic tổng hợp một số bất đẳng thức trong kì thi MO các nước

Gửi bởi Nguyenhuyen_AG trong 17-06-2015 - 13:53

Bài 55:(Ailen 2009)

Cho $a,b,c$ là các số thực thỏa $\left\{\begin{matrix} a+b+c=0\\a^2+b^2+c^2=1 \end{matrix}\right.$

Chứng minh rằng

$a^2b^2c^2\leq \frac{1}{54}$

 

 

Ngoài lời giải theo con đường khai triển $(a-b)^2(b-c)^2(c-a)^2 \geqslant 0,$ anh vừa tìm được một chứng minh tự nhiên hơn (lúc sáng đang xem Argentina đá chợt nghĩ ra :D)

 

Ta thấy bài toán là hệ quả của bất đẳng thức sau

\[(a^2+b^2+c^2)^3 \geqslant  54a^2b^2c^2, \quad (2)\]

Thật vậy theo bất đẳng thức Cauchy-Schwarz ta được

\[a^2+b^2+c^2 \geqslant \frac{(a+b)^2}{2}+c^2=\frac{3c^2}{2},\]

suy ra

\[(a^2+b^2+c^2)^3 \geqslant \frac{27c^6}{8}.\]

Như vậy ta chỉ cần chứng minh

\[\frac{27c^6}{8} \geqslant 54a^2b^2c^2,\]

hay

\[c^2 \geqslant 4\left | ab \right |.\]

Áp dụng bất đẳng thức AM-GM, ta có

\[4\left | ab \right | \leqslant \left |a+b  \right |^2 =(a+b)^2 = c^2.\]

Bài toán được chứng minh.

 

Nhận xét. Nếu thay $c=-a-b$ vào $(2)$ ta được

\[[a^2+b^2+(a+b)^2]^3 \geqslant  54a^2b^2(a+b)^2.\]

Ta có

\[[a^2+b^2+(a+b)^2]^3 - 54a^2b^2(a+b)^2=2(a-b)^2(2a+b)^2(2b+a)^2 \geqslant 0.\]

Đây cũng là một lời giải cho bài toán.

 




#566318 Topic tổng hợp một số bất đẳng thức trong kì thi MO các nước

Gửi bởi Nguyenhuyen_AG trong 16-06-2015 - 23:57

Bài 109 (IMO 2006). Tìm hằng số $M$ nhỏ nhất sao cho  bất đẳng thức

\[\left | ab(a^2-b^2)+bc(b^2-c^2)+ca(c^2-a^2) \right | \leqslant M(a^2+b^2+c^2)^2,\]

luôn đúng với mọi số thực $a,\,b,\,c$ thay đổi tùy ý.




#566310 Topic tổng hợp một số bất đẳng thức trong kì thi MO các nước

Gửi bởi Nguyenhuyen_AG trong 16-06-2015 - 23:18

Bài 80 : Cho $x,y,z$ là các số thực thỏa mãn : $x+y+z=0$ và $x^{2}+y^{2}+z^{2}=6$. Tìm GTLN của : 

$P = \begin{vmatrix} (x-y)(y-z)(z-x) \end{vmatrix}$

 

Từ giả thiết ta suy ra $xy+yz+zx = -3.$ Đặt $p=x+y+z,\,q=xy+yz+zx,\,r=xyz,$ ta có

\[P^2 = (x-y)^2(y-z)^2(z-x)^2= p^2q^2-4q^3+2p(9q-2p^2)r-27r^2.\]

Do $p=0,$ và $q=-3,$ nên

\[P^2 = 54(2-r^2) \leqslant 54 \cdot 2 = 108.\]

Suy ra $P \leqslant 6\sqrt{3}.$ Đẳng thức xảy ra khi

\[ \left\{ \begin{aligned} & xyz = 0 \\& x+y+z=0 \\& {{x}^{2}}+{{y}^{2}}+{{z}^{2}}=6 \\
\end{aligned} \right.  \]

Giải hệ này ta được $(x,y,z) = \left ( 0,\sqrt{3},-\sqrt{3} \right ).$ Vậy $P_{\max} = 6\sqrt{3}.$

 

P/s. ducvipdh12 dạo này bận quá nên anh ít lên diễn đàn. GIờ đang được nghỉ gần 2 tuần nênh giành tí thời gian luyện lại bất đẳng thức, anh thây bên này có vẻ sôi nổi hơn bên Mathscope nhỉ, bên đấy vắng quá.




#566305 Topic tổng hợp một số bất đẳng thức trong kì thi MO các nước

Gửi bởi Nguyenhuyen_AG trong 16-06-2015 - 22:52

Mình xin nhận xét một vài điều như sau :

III. Bài 75 : Cách giải của nhungviekimcuong rất hay :) . Cách mình cũng không khác cách của bạn ấy nhưng mình xin phân tích bài giải của mình . 

Thực sự , khi gặp các dạng bài như dạng bài 75 , mình cố gằng tìm dấu $=$ nhưng lại không thấy do đó minh chợt nghĩ tới ý tưởng của phương pháp SS với các bđt lệch tâm bằng cách chứng minh một bđt mạnh hơn mà giá trị tại dấu $=$ là tối thiểu . Sau khi thử nhiều số thì nhận thấy với bộ cặp số $(-3,1,1)$ làm cho giá trị $VT-VP$ có vẻ như là nhỏ nhất và đẹp nhất :P . Thế nên không do dự , dựa vào kết quả thừa  mình cộng thêm vế phải một đại lượng $\frac{4}{7}(x+y+z)^{4}$ để bđt xảy ra khi (-3,1,1) và các hoán vị . Nên ta cần chứng minh : $(a+b)^{4}+(b+c)^{4}+(c+a)^{4}\geq \frac{4}{7}(a^{4}+b^{4}+c^{4}+(a+b+c)^{4})$

Tiếp nối ý tưởng của  nhungvienkimcuong  , đặt $\left\{\begin{matrix} a+b=x & & \\ b+c=y & & \\ c+a=z & & \end{matrix}\right.$

Khai triển và rút gọn (Trâu bò đấy :P ) ta được : $24(\sum x^{4})\geq 24\sum (xy)^{2}$ (luôn đúng) (Có thể phân tích sai :) )

 

Bất đẳng thức

\[(a+b)^{4}+(b+c)^{4}+(c+a)^{4}\geq \frac{4}{7}(a^{4}+b^{4}+c^{4}+(a+b+c)^{4}), \quad (2.9.1)\]

được anh Cẩn đề xuất rất lâu trên Mathlinks và được nhắc đến trong quyển Cauchy-Schwarz anh Cẩn. Ta có một chứng minh đơn giản sau đây.

 

Nếu $a+b+c=0,$ thay $c=-a-b$ vào (2.9.1) và thu gọn ta được

\[a^4+2a^3b+3a^2b^2+2ab^3+b^4\ge 0,\]

ta có

\[a^4+2a^3b+3a^2b^2+2ab^3+b^4 = (a^2+ab+b^2)^2 \ge 0.\]

Nếu $a+b+c\ne 0$, thay $(a,\, b,\, c)$ bởi $(-a,\, -b,\, -c)$ thì bất đẳng thức vẫn không thay đổi nên ta có thể giả sử $a+b+c>0,$ và chuẩn hóa cho $a+b+c=3.$

Đặt $a=x+1,\,b=y+1,\,c=z+1,$ thì $x+y+z=0$ và $x^3+y^3+z^3=3xyz,$ ta có

$$(a+b)^4+(b+c)^4+(c+a)^4 = x^4+y^4+z^4+24(x^2+y^2+z^2)-24xyz+48,$$

tương tự thì

$$a^4+b^4+c^4+(a+b+c)^4 = x^4+y^4+z^4+6(x^2+y^2+z^2)+12xyz+84.$$

Như vậy bất đẳng thức trở thành $$x^4+y^4+z^4+48(x^2+y^2+z^2)\ge 72xyz. \quad (2.9.2)$$

Giả sử $xy \ge 0$ rồi thay $z=-x-y$ vào (2.9.2), bất đẳng thức được viết lại dưới dạng

\[2(x^2+xy+y^2)^2+96(x^2+xy+y^2)+72xy(x+y) \ge 0.\]

Theo bất đẳng thức AM-GM, ta có

\[2(x^2+xy+y^2)^2+96(x^2+xy+y^2) \ge 18x^2y^2+72(x+y)^2,\]

lại có

\[x^2y^2+4(x+y)^2+4xy(x+y) = (2x+2y+xy)^2 \ge 0.\]
Đẳng thức xảy ra khi và chỉ khi $a=b=c$ hoặc $-3a=-3b=c.$ Chứng minh của chúng ta vì thế hoàn tất.




#566295 Topic tổng hợp một số bất đẳng thức trong kì thi MO các nước

Gửi bởi Nguyenhuyen_AG trong 16-06-2015 - 22:34

Bài 55:(Ailen 2009)

Cho $a,b,c$ là các số thực thỏa $\left\{\begin{matrix} a+b+c=0\\a^2+b^2+c^2=1 \end{matrix}\right.$

Chứng minh rằng

$a^2b^2c^2\leq \frac{1}{54}$

 

Bài này hồi đấy anh chế như vầy. :D

 

Xét bất đẳng thức hiển nhiên sau

\[(a-b)^2(b-c)^2(c-a)^2 \geqslant 0. \quad (1)\]

Đặt $p=a+b+c,\,q=ab+bc+ca,\,r=abc.$ Khi đó khai triển $(1)$ ta được

\[p^2q^2-4q^3+2p(9q-2p^2)r-27r^2 \geqslant 0.\]

Cho $p = a+b+c=0,$ thì

\[-4q^3-27r^2 \geqslant 0,\]

hay

\[4(ab+bc+ca)^3+27a^2b^2c^2 \leqslant 0. \quad (2)\]

Do $a+b+c=0$ nên $a^2+b^2+c^2=(a+b+c)^2-2(ab+bc+ca)=-2(ab+bc+ca),$ do đó bất đẳng thức $(2)$ trở thành

\[(a^2+b^2+c^2)^3 \geqslant 54a^2b^2c^2.\]

Cho $a^2+b^2+c^2=1,$ ta được

\[a^2b^2c^2 \leqslant \frac{1}{54}.\]

Từ đó có bài toán trên. :D




#566290 Topic tổng hợp một số bất đẳng thức trong kì thi MO các nước

Gửi bởi Nguyenhuyen_AG trong 16-06-2015 - 22:20

Bài 40:
Cho $x,y,z$ là các số thực dương thỏa mãn $x+y+z=xyz$.CMR

$(x-1)(y-1)(z-1)\leq 6\sqrt{3}-10$


Ta chỉ cần xét trường hợp $x-1 > 0,\, y-1 > 0,\, z-1>0.$ Đặt $a = x-1 > 0,\, b = y-1 > 0,\, c = z-1>0,$ khi đó $x=a+1,\,y=b+1,\,z=c+1,$ giả thiết của bài toán trở thành
\[abc+ab+bc+ca=2.\]
Đặt $t=\sqrt[3]{abc},$ áp dụng bất đẳng thức AM-GM ta có
\[2 = abc+ab+bc+ca = t^3+ab+bc+ca \geqslant t^3 + 3\sqrt[3]{{ab}\cdot{bc}\cdot{ca}}=t^3+3t^2.\]
Do $t > 0,$ nên từ bất phương trình $2 \geqslant t^3+3t^2$ ta được $t \leqslant \sqrt{3}-1,$ suy ra $t^3 \leqslant \left(\sqrt{3}-1\right)^3.$ Suy ra tiếp
\[(x-1)(y-1)(z-1) = t^3 \leqslant \left(\sqrt{3}-1\right)^3 = 6\sqrt{3}-10.\]
Bài toán được chứng minh.


#566269 $3(a^4+b^4+c^4)+a^2+b^2+c^2+6\geq 6(a^3+b^3+c^3)$

Gửi bởi Nguyenhuyen_AG trong 16-06-2015 - 20:57

Bài toán: Cho $a,b,c$ là các số thực không âm thỏa $a+b+c=3$.Chứng minh rằng:

 

$3(a^4+b^4+c^4)+a^2+b^2+c^2+6\geq 6(a^3+b^3+c^3). \quad (2.2.1)$

 

Spoiler

 

Đặt $a=x+1,\,b=y+1,\,c=z+1,$ khi đó \[x+y+z= (a-1) + (b-1) + (c-1) = 0.\] Với phép đặt này thì \[\begin{aligned} a^2+b^2+c^2&=(x+1)^2+(y+1)^2+(z+1)^2\\&=x^2+y^2+z^2+2(x+y+z)+3\\&=x^2+y^2+z^2+3, \end{aligned}\] tương tự \[\begin{aligned} a^3+b^3+c^3& = x^3+y^3+z^3+3(x^2+y^2+z^2)+3, \\ a^4+b^4+c^4& =x^4+y^4+z^4+4(x^3+y^3+z^3)+6(x^2+y^2+z^2)+3. \end{aligned}\] Bất đẳng thức (2.2.1) trở thành \[3(x^4+y^4+z^4)+6(x^3+y^3+z^3)+x^2+y^2+z^2 \ge 0. \quad (2.2.2)\] Giả sử $xy \ge 0,$ rồi thay $z=-x-y$ vào (2.2.2), ta được \[3[x^4+y^4+(x+y)^4]+6[x^3+y^3-(x+y)^3]+x^2+y^2+(x+y)^2 \ge 0,\] \[3(x^4+2x^3y+3x^2y^2+2xy^3+y^4)+x^2+xy+y^2\ge 9xy(x+y)\] \[3(x^2+xy+y^2)^2+x^2+xy+y^2\ge 9xy(x+y).\] Áp dụng bất đẳng thức AM-GM, ta có \[x^2+xy+y^2 \ge \frac{3}{4}(x+y)^2 \ge 3xy \ge 0, \quad (2.2.3)\] suy ra \[3(x^2+xy+y^2)^2+x^2+xy+y^2\ge 27x^2y^2+\frac{3}{4}(x+y)^2.\] Ta chứng minh \[9x^2y^2+\frac{(x+y)^2}{4} \ge 3xy(x+y).\] Cũng theo bất đẳng thức AM-GM, thì \[9x^2y^2+\frac{(x+y)^2}{4} \ge 2\sqrt{9x^2y^2\cdot\frac{(x+y)^2}{4}}=3xy\left | x+y \right |\ge 3xy(x+y). \quad (2.2.4)\] Đẳng thức xảy ra khi (2.2.3) và (2.2.4) trở thành đẳng thức, tức $x,\,y$ là nghiệm của hệ \[\left\{ \begin{aligned} & x=y \\ & 27x^2y^2 = \frac{3}{4}(x+y)^2 \end{aligned}\right.\] Giải hệ này ta được $x=y=0$ hoặc $x=y=\frac{1}{3},$ suy ra $a=b=c=1,$ hoặc $a=b=\frac{4}{3},\,c=\frac{1}{3}$ cùng các hoán vị. Bài toán được chứng minh.




#566074 Topic tổng hợp một số bất đẳng thức trong kì thi MO các nước

Gửi bởi Nguyenhuyen_AG trong 15-06-2015 - 23:49

  Bài 72 (IMO 2005): Cho các số thực dương $a,b,c$ thỏa mãn $abc\geq 1$. CMR:

 

      $\frac{a^5-a^2}{a^5+b^2+c^2}+\frac{b^5-b^2}{b^5+c^2+a^2}+\frac{c^5-c^2}{c^5+a^2+b^2}\geq 0$

 

Lời giải đặc biệt của kỳ thi.

 

Ta có

\[\frac{a^5-a^2}{a^5+b^2+c^2}-\frac{a^5-a^2}{a^3(a^2+b^2+c^2)}=\frac{(a-1)^2(a^2+a+1)(b^2+c^2)}{a(a^5+b^2+c^2)(a^2+b^2+c^2)} \ge 0.\]

Suy ra

\[\sum \frac{a^5-a^2}{a^5+b^2+c^2} \ge \sum\frac{a^5-a^2}{a^3(a^2+b^2+c^2)} = \sum \frac{a^2-\frac{1}{a}}{a^2+b^2+c^2} \ge \sum \frac{\sum a^2-\sum bc}{a^2+b^2+c^2} \ge 0.\]
---------------

@ducvipdh12: chào mừng sự trở lại của anh Huyện,lâu lắm mới thấy anh onl diễn đàn :))




#566064 CMR:$\sum \sqrt{\frac{a}{b+c}...

Gửi bởi Nguyenhuyen_AG trong 15-06-2015 - 23:08

Cho $a,b,c\geq 0$ và không có đồng thời hai số bằng không. Chứng minh rằng $\sqrt{\frac{a}{b+c}}+\sqrt{\frac{b}{c+a}}+\sqrt{\frac{c}{a+b}}\geq 2\sqrt{1+\frac{abc}{(a+b)(b+c)(c+a)}}$

Spoiler

 

Chuẩn hoán $ab+bc+ca=1,$ với chú ý $a(a+b)(a+c) = a(a^2+ab+bc+ca) = a(a^2+1) = a^3+a.$ Bất đẳng thức trở thành

\[\sqrt{a^3+a}+\sqrt{b^3+b}+\sqrt{c^3+c}\ge2\sqrt{a+b+c}.\]

Áp dụng bất đẳng thức Minkowski, ta có

\[\begin{aligned} \sum \sqrt{a^3+a} &=\sqrt{a^2(a+b+c)+abc}+\sqrt{b^2(a+b+c)+abc}+\sqrt{c^2(a+b+c)+abc}\\&\ge\sqrt{\left(a\sqrt{a+b+c}+b\sqrt{a+b+c}+c\sqrt{a+b+c}\right)^2+\left(\sqrt{abc}+\sqrt{abc}+\sqrt{abc}\right)^2}\\&=\sqrt{(a+b+c)^3+9abc}\end{aligned}\]

Do đó ta cần chứng minh được

\[(a+b+c)^3+9abc\ge 4(a+b+c)(ab+bc+ca).\]

Đây chính là bất đẳng thức Schur bậc $3,$ đẳng thức xảy ra khi và chỉ khi $a=b=c $ hoặc $a=b,\;c=0$ và các hoán vị. Bài toán được chứng minh.




#566062 $(a+b+c)^2(a+b)(b+c)(c+a)\geq 24(a^2+b^2+c^2)abc$

Gửi bởi Nguyenhuyen_AG trong 15-06-2015 - 23:03

 Cho các số thực $a,b,c$ không âm. Chứng minh :

$$(a+b+c)^2(a+b)(b+c)(c+a)\geq 24(a^2+b^2+c^2)abc$$ 

 

 

Yêu cầu

 

Chuẩn hóa cho $a+b+c=3,$ và đặt $a^2+b^2+c^2=3+6t^2,\; (0\le t<1).$ Bất đẳng thức cần chứng minh được viết lại như sau
\[3\left(\dfrac{1}{a}+\dfrac{1}{b}+\dfrac{1}{c}\right) \ge 1+\dfrac{24(3+6t^2)}{9},\]
hay là

\[\dfrac{1}{a}+\dfrac{1}{b}+\dfrac{1}{c}\ge 3+\dfrac{16t^2}{3}.\]

Nếu $t=0$ thì $a=b=c=1,$ bất đẳng thức hiển nhiên đúng nên ta chi cần xét $0<t<1.$ Sử dụng đẳng thức

\[\dfrac{1}{a}=\dfrac{1}{1+2t}+\dfrac{1+2t-a}{a(1+2t)},\]

ta viết bất đẳng thức trên lại như sau

\[\displaystyle \frac{3}{1+2t}+\frac{1}{1+2t}\left(\dfrac{1+2t-a}{a}+\frac{1+2t-b}{b}+\frac{1+2t-c}{c}\right)\ge 3+\frac{16t^2}{3},\]

Dễ dàng chứng minh được rằng các số $\displaystyle 1+2t-a,1+2t-b,1+2t-c$ đều không âm. Từ đó, theo bất đẳng thức Cauchy-Schwarz, ta có

\[\begin{aligned}\dfrac{1+2t-a}{a}+\dfrac{1+2t-b}{b}+\dfrac{1+2t-c}{c}&\ge \dfrac{[(1+2t-a)+(1+2t-b)+(1+2t-c)]^2}{(1+2t-a)a+(1+2t-b)b+(1+2t-c)c}\\&=\dfrac{36t^2}{(1+2t)(a+b+c)-(a^2+b^2+c^2)}\\&=\dfrac{36t^2}{3(1+2t)-(3+6t^2)}\\&=\dfrac{6t}{1-t}.\end{aligned}\]

Vậy ta chỉ cần chứng minh được

\[\dfrac{3}{1+2t}+\dfrac{6}{(1+2t)(1-t)}\ge3+\dfrac{16t^2}{3},\]

hay là

\[\dfrac{6}{(1+2t)(1-t}-\dfrac{6t}{1+2t}\ge\dfrac{16t^2}{3},\]

\[\dfrac{6^2}{(1+2t)(1-t}\ge\dfrac{16t^2}{3},\]

\[8(1+2t)(1-t)\le9.\]
Sử dụng bất đẳng thức AM-GM, ta có \[8(1+2t)(1-t)=4\cdot (1+2t)\cdot 2(1-t)\le[(1+2t)+2(1-t)]^2=9.\]

Vậy ta có điều phải chứng minh, đẳng thức xảy ra khi và chỉ khi $a=b=c$ hoặc $a=2b=2c$ cùng các hoán vị tương ứng.




#536329 Hãy phân tích số 89 thành tổng của các số nguyên dương sao cho tích của chúng...

Gửi bởi Nguyenhuyen_AG trong 05-12-2014 - 20:24

Bài này có nguồn gốc từ IMO 1976 "http://www.artofprob...1715bc#p367432"




#532463 Topic ôn luyện VMO 2015

Gửi bởi Nguyenhuyen_AG trong 09-11-2014 - 00:40

Bài 16: Cho $A,B,C$ là 3 góc của một tam giác. CMR:

 

    $\cos\frac{A-B}{2}+\cos\frac{B-C}{2}+\cos\frac{C-A}{2}\geq \sin\frac{3A}{2}+\sin\frac{3B}{2}+\sin\frac{3C}{2}$.

 

Đây là đề thi TST của USA năm 2002, duới đây là một lời giải của Maverick

 

Let $p$ denote the semiperimeter of $\triangle ABC.$ WLOG assume $a \geq b \geq c.$
\[\cos\frac{A-B}2 =\cos\frac{A}2\cos\frac{B}2+\sin\frac{A}2\sin\frac{B}2 =  \sqrt{\frac{p^2(p-a)(p-b)}{abc^2}} +  \sqrt{\frac{(p-a)(p-b)(p-c)^2}{abc^2}}\]
\[\implies \sum \cos\frac{A-B}2= \sum \frac{a+b}c \sin\frac{C}2\]
Hence the inequality is equivalent to
\[\sum \frac{a+b}c\sin\frac{C}2 \geq 3\sum \sin\frac{A}2-4\sum \sin^3\frac{A}2\]
\[\iff \sum \frac{(b-c)(a-c)(a+b+c)}{abc} \sin\frac{C}2 \geq 0\]
\[\iff \sum (c-b)(c-a) \sin\frac{C}2 \geq 0\]
Due to assumption we have $(a-c)(b-c)\sin\frac{C}2 \geq 0$
So, it suffices to show that
\[(b-a)(c-a)\sin\frac{A}2+(a-b)(c-b)\sin\frac{B}2 \geq 0\]
\[\iff (a-b)\left((a-c)\sin\frac{A}2-(b-c)\sin\frac{B}2\right) \geq 0\]
which is obviously true.
 




#532409 [Chuẩn hóa] $\frac{(2a+b+c)^2}{2a^2+(b+c)^2}+...

Gửi bởi Nguyenhuyen_AG trong 08-11-2014 - 21:01

dạ,hình như thuần nhất thôi là được đúng không ạ,do em từng đọc tài liệu thì nó ghi là dành cho bđt đối xứng và hoán vị nhưng trong quá trình làm thì em thấy chỉ cần bđt thuần nhất thôi là được

 

NTP

Chỉ cần với bất đẳng thức thuần nhất là ta có thể chuẩn hóa được. Chuẩn hóa là một kỹ thuật khá quan trọng đặc biệt là với trong phương pháp U.C.T. Tuy nhiên có một số đã hiểu sai và to tát quá kỹ thuật này, thậm chí với số bạn rất giỏi bất đẳng thức nhưng không hiểu rõ được bản chất của chuẩn hóa. Ta có thể hiểu chuần hóa là phép đặt ẩn phụ.




#532402 $\sum cos\frac{A-B}{2}\geq \sum...

Gửi bởi Nguyenhuyen_AG trong 08-11-2014 - 20:32

   Cho $A,B,C$ là 3 góc của một tam giác. CMR :

 

      $cos\frac{A-B}{2}+cos\frac{B-C}{2}+cos\frac{C-A}{2}\geq sin\frac{3A}{2}+sin\frac{3B}{2}+sin\frac{3C}{2}$

 

 

 

 P/s: Bài này khá hay

USA TST 2003: http://www.artofprob...hp?f=54&t=3911